Proving an Inequality with Elementary Methods

  • Thread starter DorelXD
  • Start date
  • Tags
    Inequality
In summary, the problem is to prove the inequality (1+x_1^21^2)(1+x_2^22^2)...(1 + x_n^2n^2) ≥ \frac{2n^2+9n+1}{6n} for 0 ≤ x_1 ≤ x_2 ≤ ... ≤ x_n and x_1 + x_2 + ... + x_n = 1, where all the 'x' are real and n is a natural number. The attempts at a solution include checking the field, using known inequalities such as the average mean and Cauchy-Buniakowski-Schwarz, and solving the optimization problem with numerical packages. It
  • #1
DorelXD
126
0

Homework Statement



Let [itex] 0 ≤ x_1 ≤ x_2 ≤ ... ≤ x_n [/itex] and [itex] x_1 + x_2 + ... + x_n = 1 [/itex] , All the 'x' are real and n is a natural number. Prove the following:

[tex] (1+x_1^21^2)(1+x_2^22^2)...(1 + x_n^2n^2) ≥ \frac{2n^2+9n+1}{6n} [/tex]


Homework Equations




The Attempt at a Solution



Well...all I managed to do was to "check the field a little". First of all, the right member of the inequality somehow reminds me of the sum:

[tex] 1^2 + 2^2 + ... + n^2 = \frac{n(n+1)(2n+1)}{6} = \frac{(n+1)(2n^2 + 3n + 1)}{6} [/tex]

I know from experience that we can solve this type of exercises by using some well-know inequalities such as the one that relates the average mean with the geometric one, or the Cauchy-Buniakowski-Schwarz ( C-B-S ). So:

[tex]1 ≥\frac{1}{n} ≥ \sqrt[n]{x_1x_2...x_n} [/tex] , so it follows immediately that : [tex] x_1x_2...x_n ≤ 1[/tex]

That's all I could do. Could you help me continue, please? This exercise has been stuck in my head for two weeks already. I can't solve it alone, and I would like that somebody could guide me to the solution. Thank you!
 
Last edited:
Physics news on Phys.org
  • #2
DorelXD said:

Homework Statement



Let [itex] x_1 ≤ x_2 ≤ ... ≤ x_n [/itex] and [itex] x_1 + x_2 + ... + x_n = 1 [/itex] , All the 'x' are real and n is a natural number. Prove the following:

[tex] (1+x_1^21^2)(1+x_2^22^2)...(1 + x_n^2n^2) ≥ \frac{2n^2+9n+1}{6n} [/tex]


Homework Equations




The Attempt at a Solution



Well...all I managed to do was to "check the field a little". First of all, the right member of the inequality somehow reminds me of the sum:

[tex] 1^2 + 2^2 + ... + n^2 = \frac{n(n+1)(2n+1)}{6} = \frac{(n+1)(2n^2 + 3n + 1)}{6} [/tex]

I know from experience that we can solve this type of exercises by using some well-know inequalities such as the one that relates the average mean with the geometric one, or the Cauchy-Buniakowski-Schwarz ( C-B-S ). So:

[tex]1 ≥\frac{1}{n} ≥ \sqrt[n]{x_1x_2...x_n} [/tex] , so it follows immediately that : [tex] x_1x_2...x_n ≤ 1[/tex]

That's all I could do. Could you help me continue, please? This exercise has been stuck in my head for two weeks already. I can't solve it alone, and I would like that somebody could guide me to the solution. Thank you!

Are all the ##x_i \geq 0?##
 
  • #3
Yes, they are. I also modified the first post, by adding this fact.
 
  • #4
Can you help me ? Do you have any idea :D ?
 
  • #5
This is under precalculus. If calculus methods are permitted I would use Lagrange multipliers.
 
  • #6
You can make elementary (non calculus) arguments that the left side of the inequality is ≥ (1+n). Then, you can get the rest by algebra.
 
  • #7
DorelXD said:
Yes, they are. I also modified the first post, by adding this fact.

One can solve the optimization problem
[tex] \min \sum_{i-1}^n \log(1 + i^2 x_i^2) \\
\text{subject to}\\
x_1 + x_2 + \cdots + x_n = 1 \\
0 \leq x_1 \leq x_2 \leq \cdots \leq x_n[/tex]
using a numerical optimization package for various small to medium values of n (3 ≤ n ≤ 10). In all cases I tried the optimal solution is ##x_1 = x_2 = \cdots = x_n = 1/n##. Furthermore, this can be shown to be a so-called Kuhn-Tucker point for the problem with general n. The minimization objective is pseudo-convex and the constraint set is convex, so satisfying the Karush-Kuhn-Tucker conditions is, I believe, sufficient for a (global) minimum.

In other words, we have
[tex] (1+1^2 x_1^2)(1 + 2^2 x_2^2) \cdots (1+ n^2 x_n^2)
\geq (1 + (1/n)^2)(1+(2/n)^2) \cdots (1 + (n/n)^2)[/tex]
for all feasible x. Maybe that can be manipulated further to give the desired result.
 
  • #8
PAllen said:
This can be really simple. You have a product of terms. Are they all ≥ 1 even for xi=0? Can the nth x term value ever be < 1/n, given the other constraints? This is enough to prove the product ≥ (1+n). Then the rest follows by algebra. This is a pre-calculus problem not expected to be solved with software packages. Let's not kill a mosquito with canon.
This is confusing. 'n' is the number of terms, so the nth term means just the last term. Certainly xn >= 1/n, so the last term is at least 2. I don't see how you get it's at least 1+n.
If you meant a more generic n, I still don't see it. xj need not be ≥ 1/j, nor 1/n. Every xj except xn could be zero.
 
  • #9
haruspex said:
This is confusing. 'n' is the number of terms, so the nth term means just the last term. Certainly xn >= 1/n, so the last term is at least 2. I don't see how you get it's at least 1+n.
If you meant a more generic n, I still don't see it. xj need not be ≥ 1/j, nor 1/n. Every xj except xn could be zero.

Sorry, I realized my mistake as soon as finished, and was working on deleting my post. I missed that the x's were squared as well as the i's. Oops.
 
  • #10
Ray Vickson said:
In all cases I tried the optimal solution is ##x_1 = x_2 = \cdots = x_n = 1/n##.

Suppose xj < xj+1 for some j < n. It might be straightforward to show that substituting xj' = xj+1' = (xj + xj+1)/2 produces a lower product. (Calculus by the back door.)
 
  • #11
PAllen said:
Sorry, I realized my mistake as soon as finished, and was working on deleting my post. I missed that the x's were squared as well as the i's. Oops.
Yeah, I've been there - a frantic dash to delete a regretted post, only to find it's already been read:redface:.
 
  • #12
haruspex said:
Suppose xj < xj+1 for some j < n. It might be straightforward to show that substituting xj' = xj+1' = (xj + xj+1)/2 produces a lower product. (Calculus by the back door.)

Unfortunately, this is not true. The substitution produces a larger product. Larger by the difference between xj and its successor, over 2, squared.
 
  • #13
PAllen said:
Unfortunately, this is not true. The substitution produces a larger product. Larger by the difference between xj and its successor, over 2, squared.
Hmm.. I just tried it and it worked for me. I ended up needing that 1 > j(j+1)xjxj+1, which is clearly true.
 
  • #14
haruspex said:
Hmm.. I just tried it and it worked for me. I ended up needing that 1 > j(j+1)xjxj+1, which is clearly true.

A much easier way to get a lower bound is to note that for positive ##y_i = i^2 x_i^2## we have
[tex] (1+y_1)(1+y_2) \cdots (1+y_n) > 1 + \sum_{i=1}^n y_i, [/tex]
because the RHS is just the first two terms in the expansion of the product, and all the omitted terms are > 0. Therefore, we can get a lower bound by solving the relatively simple problem
[tex] \min \sum_{i=1}^n i^2 x_i \:\:\:\leftarrow \sum_{i=1}^n y_i\\
\text{subject to}\\
\sum_{i=1}^n x_i = 1\\
0 \leq x_1 \leq x_2 \leq \cdots \leq x_n[/tex]
This problem has the optimal solution ##x_1 = x_2 = \cdots = x_n = 1/n##, so we have
[tex] (1+1^2 x_1^2)(1+2^2 x_2^2) \cdots (1 + n^2 x_n^2) > 1 + \sum_{i=1}^n i^2/n^2 [/tex]
for all feasible ##x_1, x_2, \ldots x_n##.
 
Last edited:
  • #15
haruspex said:
Hmm.. I just tried it and it worked for me. I ended up needing that 1 > j(j+1)xjxj+1, which is clearly true.

I keep forgetting the squaring.
 
Last edited:
  • #16
Ray Vickson said:
A much easier way to get a lower bound is to note that for positive ##y_i = i^2 x_i^2## we have
[tex] (1+y_1)(1+y_2) \cdots (1+y_n) > 1 + \sum_{i=1}^n y_i, [/tex]
because the RHS is just the first two terms in the expansion of the product, and all the omitted terms are > 0. Therefore, we can get a lower bound by solving the relatively simple problem
[tex] \min \sum_{i=1}^n i^2 x_i \:\:\:\leftarrow \sum_{i=1}^n y_i\\
\text{subject to}\\
\sum_{i=1}^n x_i = 1\\
0 \leq x_1 \leq x_2 \leq \cdots \leq x_n[/tex]
This problem has the optimal solution ##x_1 = x_2 = \cdots = x_n = 1/n##, so we have
[tex] (1+1^2 x_1^2)(1+2^2 x_2^2) \cdots (1 + n^2 x_n^2) > 1 + \sum_{i=1}^n i^2/n^2 [/tex]
for all feasible ##x_1, x_2, \ldots x_n##.

This seems promising. This lower bound appears to equal the RHS. If this could be proven (along with the rest), you'd be done.
 
  • #17
Ray Vickson said:
A much easier way to get a lower bound is to note that for positive ##y_i = i^2 x_i^2## we have
[tex] (1+y_1)(1+y_2) \cdots (1+y_n) > 1 + \sum_{i=1}^n y_i, [/tex]
because the RHS is just the first two terms in the expansion of the product, and all the omitted terms are > 0. Therefore, we can get a lower bound by solving the relatively simple problem
[tex] \min \sum_{i=1}^n i^2 x_i \:\:\:\leftarrow \sum_{i=1}^n y_i\\
\text{subject to}\\
\sum_{i=1}^n x_i = 1\\
0 \leq x_1 \leq x_2 \leq \cdots \leq x_n[/tex]
This problem has the optimal solution ##x_1 = x_2 = \cdots = x_n = 1/n##, so we have
[tex] (1+1^2 x_1^2)(1+2^2 x_2^2) \cdots (1 + n^2 x_n^2) > 1 + \sum_{i=1}^n i^2/n^2 [/tex]
for all feasible ##x_1, x_2, \ldots x_n##.

We need the proof for this in elementary terms.
If this is proven, the problem is basically solved.

Edit: I just saw that the [itex]x_i[/itex] had to be positive. Then, the proof is obviously very easy.
The sum [itex]\displaystyle \sum_{i=1}^n i^2/n^2[/itex] has closed form [itex]\displaystyle \frac{(n+1)(2n+1)}{6n}[/itex]. The inequality then follows from the optimal solution.
 
Last edited:

What is an inequality to prove?

An inequality to prove is a mathematical statement that compares two quantities or expressions using the symbols <, >, ≤, or ≥. The goal is to show that one quantity is either greater than or less than the other, or that they are equal.

Why is it important to prove inequalities?

Proving inequalities is important in mathematics because it allows us to make accurate and precise comparisons between quantities. It also helps us to understand the relationships between different mathematical expressions.

What are some common techniques for proving inequalities?

Some common techniques for proving inequalities include using algebraic manipulation, substitution, and mathematical induction. Other strategies may include using the properties of inequalities, such as the transitive property or the triangle inequality.

How can I determine if an inequality is true?

To determine if an inequality is true, you can use algebraic methods or numerical examples. You can also graph the two sides of the inequality on a coordinate plane and see if they intersect at any point, which would indicate that the inequality is not true.

What are some real-life applications of proving inequalities?

Proving inequalities has many real-life applications, such as in economics, where it is used to analyze supply and demand, and in physics, where it is used to determine relationships between physical quantities. It is also commonly used in optimization problems in engineering and business.

Similar threads

  • Precalculus Mathematics Homework Help
Replies
10
Views
2K
  • Precalculus Mathematics Homework Help
Replies
14
Views
1K
  • Precalculus Mathematics Homework Help
Replies
3
Views
945
  • Precalculus Mathematics Homework Help
Replies
3
Views
950
  • Precalculus Mathematics Homework Help
Replies
11
Views
2K
Replies
1
Views
571
  • Precalculus Mathematics Homework Help
Replies
13
Views
1K
  • Precalculus Mathematics Homework Help
2
Replies
53
Views
5K
Replies
3
Views
732
  • Calculus and Beyond Homework Help
Replies
1
Views
704
Back
Top